Đến nội dung

Ngockhanh99k48

Ngockhanh99k48

Đăng ký: 20-10-2015
Offline Đăng nhập: 12-01-2018 - 17:03
****-

#667733 VMF's Marathon Hình học Olympic

Gửi bởi Ngockhanh99k48 trong 09-01-2017 - 14:47

$\boxed{\text{Lời giải bài toán 106}}$
Gọi $M$ là tâm nội tiếp $\triangle BDF$. Ta sẽ chứng minh $M \in (IAB)$. Ta có $\triangle FBD \cap M \stackrel{+}{\sim} \triangle FEA \cap I$, do đó $\widehat{AIM}=\widehat{AIF}+\widehat{FIM} = 90^{\circ}+\frac{\widehat{AEF}}{2}+\widehat{FAD}=180^{\circ}-\frac{\widehat{ABC}}{2} = 180^{\circ} - \widehat{ABM}$ nên $A, I, M, B$ đồng viên. Tương tự ta sẽ suy ra $N$ là tâm nội tiếp $\triangle CDE$.
Gọi $K, L$ là trung điểm $HB, HC$. Kẻ đường kính $EP$ của $(CDE)$. Rõ ràng $EP$ đi qua $L$. Ta có $\triangle DBF \cap KMH \stackrel{+}{\sim} \triangle DEC \cap LNP$. Gọi $Q, O$ là tâm ngoại tiếp $\triangle DEF, \triangle ABC$ thì $Q$ là trung điểm $OH$. Ta có $\frac{1}{2} (QK, QL) \equiv \frac{1}{2} (OB, OC) \equiv (AB, AC)$ $\equiv$ $(HC, HE) \equiv (EH, EL) \equiv (EH, KM)+(KM, LN)+(LN, EL) \equiv (KM, LN)$ (mod $\pi$). Biểu thức cuối đúng do $\triangle BKM \stackrel {+}{\sim} \triangle ELN$. Từ đó ta suy ra $KM$ cắt $LN$ thuộc $(Q)$.
$\boxed{\text{Bài toán 107}}$ (Nguyễn Minh Hà) Cho tam giác ABC không cân tại $A$, $BB', CC'$ là các đường cao. $M, N$ thứ tự là trung điểm $AC, AB$. $E, F$ thứ tự là tâm nội tiếp $(I)$ và $AC, AB$. $P, Q$ theo thứ tự là giao điểm của $BB', CC'$ và $MI, NI$. $L$ là giao điểm của $EF$ và $PQ$. Chứng minh $IL \parallel BC$.


#665976 Tuần 4 tháng 12/2016 : Bài toán chia đôi cạnh

Gửi bởi Ngockhanh99k48 trong 27-12-2016 - 08:55

Một lời giải sử dụng phép nghịch đảo:
Xét phép nghịch đảo cực $A$ phương tích $AB.AC$ hợp với phép đối xứng trục phân giác góc $A$, ta chuyển về bài toán sau:
Cho tam giác $ABC$ nội tiếp đường tròn $(O)$, đường tròn bàng tiếp góc $A$ là $(I_a)$ tiếp xúc $BC$ tại $D$. $(AI_a), (AD)$ cắt $(O)$ tại điểm thứ hai là $E, F$. $AD$ cắt $(AI_a)$ tại $G$. Đường tròn qua $A, G$ có tâm là $H$ thỏa mãn $AH \perp BC$ cắt $(O)$ tại $K$.$(AD)$ cắt $(AI_a)$ tại điểm thứ hai $N$. $AN$ cắt $(O)$ tại $P$. Chứng minh tứ giác $PEKF$ điều hòa.
Lời giải:
Gọi $L,M, Q$ là trung điểm $AD, BC, AI_a$. Dễ thấy $AN \parallel BC$. Ta có $OM \perp AP, OL \perp AF, OH \perp AK, OQ \perp AE$ nên tứ giác $PEKF$ điều hòa khi và chỉ khi $(OM, OH, OQ, OL)=-1$. Tương đương với $OH$ chia đôi $LQ$ do $LQ \perp BC$. Vị tự tâm $A$ tỉ số 2, ta có bài toán:
Cho tam giác $ABC$ đường tròn bàng tiếp góc $A$ là $(I)$ tiếp xúc $BC$ tại $D$. $P$ là điểm thỏa mãn $AP \perp BC$, $IP \perp AD$. Kẻ đường kính $AQ$ của $(ABC)$. Khi đó $PQ$ chia đôi $ID$. Bài toán trên là bài toán của thầy trên đường tròn bàng tiếp góc $A$.


#665895 Tuần 4 tháng 12/2016 : Bài toán chia đôi cạnh

Gửi bởi Ngockhanh99k48 trong 26-12-2016 - 14:31

Lời giải:
Trước tiên ta làm một số bài toán sau:
Bài toán 1: Cho $\triangle ABC$, đường tròn mixtilinear nội tiếp và bàng tiếp góc $\widehat{BAC}$ của $\triangle ABC$ là $(I_a)$ và $(J_a)$ tiếp xúc $(ABC)$ thứ tự tại $P, Q$. $PQ$ cắt $BC$ tại $E$. Khi đó $AE$ tiếp xúc $(BAC)$.

Post 380.PNG

Chứng minh:
Xét phép nghịch đảo cực $A$ phương tích $AB.AC$ hợp với phép đối xứng trục phân giác góc $\widehat{BAC}$:
$I_{A}^{AB.AC} \circ Đ_{l} : X \Leftrightarrow X' $.
Khi đó $C \equiv B', B \equiv C'$. $P', Q'$ thứ tự là tiếp điểm của đường tròn bàng tiếp góc $A$ và nội tiếp của $\triangle ABC$ với đường thẳng $BC$. Khi đó $P', Q'$ đối xứng nhau qua trung trực $BC$. Qua $A$ kẻ đường thẳng song song với $BC$ cắt $(ABC)$ tại điểm thứ hai $R$ thì $R \in (AP'Q')$. Do đó $R \equiv E'$. $AR, AE$ đẳng giác nên $AE$ là tiếp tuyến của $(ABC)$.
Bài toán 2: Cho tam giác $ABC$ nội tiếp $(O)$. Đường tròn mixtilinear nội tiếp góc $A$ là $(I_a)$ của $\triangle ABC$ tiếp xúc $AB, AC$ và $(O)$ tại $D, E, F$. $AF$ cắt $DE$ tại $L$. Đường thẳng qua $L$ vuông góc $OA$ cắt $BC$ tại $G$. Khi đó $FG$ tiếp xúc $(O)$.

Post 381.PNG

Chứng minh:
$DE$ cắt $BC$ tại $K$. Gọi $I$ là tâm nội tiếp $\triangle ABC$. $AH$ là đường cao tam giác $ABC$. Ta có $(LG, LK) \equiv (AO, AI_a) \equiv (AI, AH) \equiv (KL, KG)$ (mod $\pi$). Do đó $\triangle GLK$ cân tại $G$. Hơn nữa ta có tính chất: $AI$ cắt $FK$ tại $J$ là điểm chính giữa cung $BC$ không chứa $A$ của tam giác $ABC$. Ta có $(FK, FA) \equiv (FJ, FA) \equiv (BJ, BA) \equiv (BJ, BC) + (BC, BA) \equiv (AJ, AC ) + \frac{\pi}{2} - (OA, AC) \equiv (AJ, AO) + \frac{\pi}{2} \equiv \frac{\pi}{2} - (LG, LK)$ (mod $\pi$). Do đó $G$ là tâm $(DKF)$. Suy ra $(FG, FA) \equiv \frac{\pi}{2} - (DE, FJ) \equiv \frac{\pi}{2} - (DE, AJ) + (JF, JA) \equiv (CF, CA)$. Do vậy $GF$ là tiếp tuyến của $(O)$.
Quay lại bài toán:

Post 382.PNG

$IM$ cắt $DN$ tại $X$. $(K)$ tiếp xúc $AB, AC$ tại $E, F$. $A, I, D, X$ đồng viên và $B, E, I, D$ đồng viên nên $(AX, AD) \equiv (IX, ID ) \equiv (BA, BD) \equiv (CA, CD)$ (mod $\pi$) nên $AX$ là tiếp tuyến của $(O)$. Do đó $AX \parallel QR$. $AX$ cắt $BC$ tại $Y$. Gọi $(S)$ là đường tròn mixtilinear bàng tiếp góc $A$ của $\triangle ABC$, tiếp xúc $(O)$ tại $T$. Theo bài toán 1 thì $Y, D, T$ thẳng hàng. Kẻ đường kính $AZ$ của $(O)$. Thế thì $Z$ thuộc $DN$. Gọi $U, V$ là điểm chính giữa của cung $BC$ thứ tự có và không chứa điểm $A$. Theo bài toán 2 thì $DP$ tiếp xúc $(O)$. Một tính chất của đường tròn mixtilinear là $DI$ và $I_aT$ cắt nhau tại $U$, với $I_a$ là tâm bàng tiếp góc $A$ của $\triangle ABC$. Ta có $UZ \parallel AI$ và $V$ là trung điểm $II_a$ nên $(DTVZ) = U(DTVZ) = U(II_aVZ)=-1$. Do đó $(PYMN)=D(PYMN)=D(DTVZ)=-1$. Suy ra $A(PYQR)=A(PYMN)=-1$. Do $AY \parallel QR$ nên $P$ là trung điểm $QR$. Ta có đpcm.




#664475 Tuần 2 tháng 12/2016 : Bài toán nội tiếp trên đường tròn tiếp xúc

Gửi bởi Ngockhanh99k48 trong 12-12-2016 - 18:22

Lời giải:
$ID^2=\overline{IB}.\overline{IN}=\overline{IC}.\overline{IM}$. Do đó $B, N, M, C \in (J)$. $X=MN \cap BC, Y=BM \cap CN$. Dễ thấy $(DMN)$ tiếp xúc $(I)$ nên $\overline{XB}.\overline{XC}=\overline{XM}.\overline{XN}=XD^2$ nên $X$ thuộc trục đẳng phương của $(I)$ và $(J)$. Theo định lí Brocard ta có $XY \perp IJ$ và $IX \perp JY$ tại $U$. Hơn nữa ta cũng có $U \in (BIC)$ nên $\overline{XU}.\overline{XI}=\overline{XB}.\overline{XC}=XD^2$ suy ra $\widehat{XUD}=\widehat{XDI}=90^{\circ}$. Do đó $D \in JY$.
$XY \perp IJ$ nên $Y$ thuộc trục đẳng phương của $(I)$ và $(J)$. Nếu $DY$ cắt $(I)$ tại $V$ thì dễ thấy $XV$ tiếp xúc $(BVC)$ ($VD$ là đối cực của $X$ trong $(I)$), $(I)$ và $V \in (BDM), (CDN)$. Gọi $S'=VE \cap BM, T'=VF \cap CN$.
Ta chỉ cần chứng minh $S', T' \in (BVC)$ và $S'T' \equiv PQ$ là xong. Ta có $VB, VN$ đẳng giác trong góc $\widehat{FVD}$ nên $\widehat{T'VB}=\widehat{DVN}=\widehat{BCT'}$, suy ra $T' \in (BVC)$. Tương tự $S' \in (BVC)$.
$\widehat{MES'}=\widehat{VFN}, \widehat{EMS'}=\widehat{BMD}=\widehat{BVD}=\widehat{FVN}$ nên $\triangle MES' \sim \triangle VFN$. Do đó $\frac{ES'}{EP}=\frac{2ES'}{EM}=\frac{2FN}{FV}=\frac{FD}{FV}$. Suy ra $\triangle FDV \sim \triangle ES'P$. Do đó $\widehat{FEV}=\widehat{FDV}=\widehat{ES'P}$, suy ra $PS' \parallel EF$ hay $S' \in PQ$ do $PQ \parallel EF$. Tương tự $T' \in PQ$. Ta có đpcm.


#663832 Tuần 1 tháng 12/2016 : Bài toán vuông góc trên cấu hình tiếp xúc

Gửi bởi Ngockhanh99k48 trong 04-12-2016 - 23:08

Bài toán tổng quát:
Cho tam giác $\triangle ABC$ với $P, Q$ là hai điểm liên hợp đẳng giác nằm trên phân giác góc $\widehat{BAC}$. Gọi $\triangle P_aP_bP_c$ và $\triangle Q_aQ_bQ_c$ là tam giác pedal ứng với hai điểm $P, Q$ của $\triangle ABC$. Gọi $X, Y$ là hình chiếu của $P_a, Q_a$ trên $Q_bQ_c, P_bP_c$. Kẻ $P_aP_a'$ và $Q_aQ_a'$ vuông góc với $PQ$. $P_a'Y$ và $Q_a'X$ cắt $(P_aP_bP_c)$ tại điểm $M, N$. Khi đó $B, M, N, C$ đồng viên.


#663830 VMF's Marathon Hình học Olympic

Gửi bởi Ngockhanh99k48 trong 04-12-2016 - 22:45

$\boxed{\text{Lời giải bài toán 96}}$
Đường qua $H$ vuông góc $HN$ cắt $DC$, $HB$ tại $L, K$. Đường qua $H$ vuông góc $HM$ cắt $DB$, $HC$ tại $E, F$. $KL$ cắt $EF$ tại $H'$. Dễ thấy $H, O, H'$ thẳng hàng. Vậy ta chỉ cần chứng minh $H, D, H'$ thẳng hàng. Ta có $\widehat{EFC}=\widehat{HMB}=\widehat{EBC}$ nên $E, F, B, C$ đồng viên. Ta có $\widehat{LKB}=\widehat{HNC}=\widehat{LCB}$ nên $L, K, C, B$ đồng viên. $HB, HC$ cắt $AC, AB$ tại $P, Q$.
Mặt khác $\triangle HMF \sim \triangle HNK$ nên $\frac{HQ}{HP}=\frac{HF}{HK}$ nên $PQ \parallel KF$. Như vậy $B, F, K, C$ đồng viên. Do đó 6 điểm $B, F, L, E, K, C$ đồng viên. Áp dụng định lí Pascal cho 6 điểm trên ta có $H, D, H'$ thẳng hàng. Ta có đpcm.


#663807 Tuần 1 tháng 12/2016 : Bài toán vuông góc trên cấu hình tiếp xúc

Gửi bởi Ngockhanh99k48 trong 04-12-2016 - 18:19

Một lời giải khác:
Tiếp tuyến tại $P$ cắt $BC$ tại $X$. Gọi $Y$ là đối xứng của $D$ qua $X$. Ta có $XD^2=XP^2=XY^2=\overline{XB}.\overline{XC}$. Do đó $(YDBC)=-1$ và $Y$ thuộc $EF$. Từ đó ta có $Y, P, K, D$ cùng thuộc đường tròn đường kính $YD$. Ta có $\widehat{PLD}+\widehat{DKL}=\widehat{PDY}+\widehat{PYD}=90^{\circ}$ hay là $DL \perp DK$. Do $DK \perp EF$, $EF \perp AI$ nên $DK \parallel AI$. Ta có đpcm.


#662582 Tuần 4 tháng 11/2016 : Trục đẳng phương đi qua trực tâm

Gửi bởi Ngockhanh99k48 trong 20-11-2016 - 23:39

Lời giải :
Bài toán trên là sự kết hợp của hai bài toán con như sau:
Bài 1: Cho $\triangle LBC$ với trọng tâm $A$. Gọi $H$ là trực tâm $\triangle ABC$ và $HB, HC$ thứ tự cắt $AC, AB$ tại $E, F$. Khi đó $EF$ là trục đẳng phương của $(ABC)$ và $(LH)$.
Lời giải: Kẻ $LK \perp HB$, tức là $LK \perp CA$. Do $CA$ chia đôi $LB$ nên $F$ là trung điểm $KB$. Do đó $\mathbb{P}_{F/(ABC)}= \overline{FA}.\overline{FC}=-\overline{FB}.\overline{FH}=\overline{FK}.\overline{FH}=\mathbb{P}_{F/(LH)}$. Do đó $F$ thuộc trục đẳng phương của $(ABC)$ và $(LK)$. Tương tự xét điểm $E$ ta có đpcm.
Bài 2: Cho $\triangle ABC$ với trực tâm $H$ và nội tiếp $(O)$. $BH, CH$ cắt $AC, AB$ tại $E, F$. Gọi $d$ là đường thẳng qua $H$ song song $BC$. $EF$ cắt $d$ tại $L$. Gọi $K$ là hình chiếu của $L$ trên $OH$. Khi đó $H$ thuộc trục đẳng phương của đường tròn $(OK)$ và đường tròn Euler của $\triangle ABC$.
Lời giải: Gọi $M$ là trung điểm $BC$. $OM$ cắt $d$ tại $N$. Do đó $\overline{HO}.\overline{HK}=\overline{HL}.\overline{HN}$(1). Lấy $P$ thuộc $(O)$ sao cho $AP \parallel BC$. Kẻ $PQ \perp BC$ với $Q \in d$. Khi đó $BHQC$ là hình thang cân nên $Q \in (BHC)$. Xét phép nghịch đảo cực $H$, phương tích $\overline{HE}.\overline{HB}$: $\mathbb{I}_{H}^{\overline{HE}.\overline{HB}}$ biến $B \leftrightarrow E$, $C \leftrightarrow F$, $d \leftrightarrow d$. Do đó $L=EF \cap d \leftrightarrow (BHC) \cap d$. Suy ra $L, Q, E, B$ đồng viên. Suy ra $\overline{HN}.\overline{HL}=\frac{1}{2} \overline{HL}.\overline{HQ}=\frac{1}{2}.\overline{HE}.\overline{HB} = \mathbb{P}_{H/(Euler)}$(2). Từ (1) và (2) ta có đpcm.


#660101 Tuần 1 tháng 11/2016 : Trục đẳng phương đi qua giao điểm

Gửi bởi Ngockhanh99k48 trong 31-10-2016 - 20:45

Một hướng đi gần giống:
$AK$ cắt $(O)$ tại $E$. $PK$ cắt $(BOC)$ tại $F$. Ta có $\overline{KE}.\overline{KA}=\overline{KB}.\overline{KC}=\overline{KP}.\overline{KF}$ nên $A, P, E, F$ đồng viên. Do đó $\widehat{PAK}=\widehat{PFA}$ (vì $PA=PE$). Suy ra $PA$ là tiếp tuyến của $(AKF)$.
$AP$ cắt $(BOC)$ tại $Q$. $QF, PE$ cắt $BC$ tại $M, N$. Do $AK \parallel PT$ nên $PT$ là phân giác của góc $\widehat{EPQ}$. Nếu $PE$ cắt $(BOC)$ tại $S$ thì $SQ \parallel BC$. Bằng cộng các cung chứa góc của đường tròn $(BOC)$ suy ra $\widehat{PFQ}=\widehat{ENM}$ hay $M, F, N, P$ đồng viên. Suy ra $\widehat{QMN}=\widehat{FPE}=\widehat{FAK}$ do đó $A, K, F, M$ đồng viên.
$(AKM)$ cắt $AD$ tại $Z$ thì ta có $\widehat{KLD}=\widehat{KAP}=\widehat{AMK}=\widehat{KZD}$ nên $Z \in (KLD)$. Do đó $\widehat{ZFP}=\widehat{DAK}=\widehat{ZDP}$ hay $Z, F, P, D$ đồng viên. Đến đây ta có đpcm.


#659150 Tuần 4 tháng 10/2016: Đường tròn tiếp xúc đường tròn cố định

Gửi bởi Ngockhanh99k48 trong 23-10-2016 - 23:21

Bài toán trên theo em nghĩ có thể là trường hợp đặc biệt của bài tổng quát sau: Cho $\triangle ABC$ với đường tròn $(K)$ bất kì đi qua $B, C$. Gọi $X, Y$ là hai điểm bất kì chạy trên $(O)$ và $(K)$. $(BXY)$ và $(CXY)$ cắt tiếp tuyến tại $X$ của $(O)$ tại $S, T$. Chứng minh $(STY)$ tiếp xúc $(K)$


#659147 Tuần 4 tháng 10/2016: Đường tròn tiếp xúc đường tròn cố định

Gửi bởi Ngockhanh99k48 trong 23-10-2016 - 23:06

Lời giải:
$(PCE)$ cắt $(PBF)$ tại điểm thứ hai $S$. Gọi $H$ là trực tâm $\triangle ABC$. Do $\triangle PCE$ đồng dạng $\triangle AOB$ và $\triangle PBF$ đồng dạng $\triangle AOC$ nêm cộng góc ta có $\widehat{EPF}=\widehat{BPF}+\widehat{BPC}+\widehat{CPE}=\widehat{BPC}+\widehat{BAC}=180^{\circ}$. Do đó $E, P, F$ thẳng hàng. Do đó $S$ là tâm vị tự quay biến $EM$ thành $FN$. Mặt khác $S$ là điểm Miquel của tứ giác toàn phần $QFPMNE$ nên $S$ thuộc $(QMN)$ và $(QEF)$. Ta lại có $\widehat{BSC}=\widehat{BAC}$ và $S$ nằm khác phía $A$ so với $BC$ nên $S$ thuộc $(BHC)$. Từ giờ kẻ tiếp tuyến tại $S$ và cộng góc ta dễ suy ra $(QMN)$ tiếp xúc $(BHC)$ tại $S$.


#658766 Chứng minh rằng: $c^2<2a^2+b^2$

Gửi bởi Ngockhanh99k48 trong 22-10-2016 - 11:47

$\widehat{C} = 2\widehat{A}+\widehat{B}$ nên $2\widehat{C}=180^{\circ}+\widehat{A}$ hay góc $C$ tù. Do đó $\cos C <0$. Bạn chứng minh đoạn cuối chưa chắc vô lí.


#658176 Chứng minh rằng $\frac{1}{a^{2}+b^{2}+c^{2}}+\frac{1}{\sq...

Gửi bởi Ngockhanh99k48 trong 17-10-2016 - 11:07

Sử dụng AM-GM ta có:
$\frac{1}{a^2+b^2+c^2} + \frac{1}{3\sqrt{abc}} +\frac{1}{3\sqrt{abc}} +\frac{1}{3\sqrt{abc}} \geq 4 \sqrt[4] {\frac{1}{a^2+b^2+c^2}.(\frac{1}{3\sqrt{abc}})^3} = 4 \sqrt[4]{\frac{1}{27(a^2+b^2+c^2)\sqrt{a^3b^3c^3}}}$
Cuối cùng ta chứng minh $27abc(a^2+b^2+c^2)\sqrt{abc} \leq 81$.
Theo AM-GM ta có $abc \leq 1$. Do đó
$27abc(a^2+b^2+c^2)(abc)^{\frac{1}{3}}.(abc)^{\frac{1}{6}}$ $\leq$ $9abc(a^2+b^2+c^2)(a+b+c)$ $\leq$ $3(a^2+b^2+c^2)(ab+bc+ca)^2$ $\leq$ $\frac{1}{9}[(a^2+b^2+c^2)+2(ab+bc+ca)]^3 = \frac{(a+b+c)^6}{9} =81$. Ta có đpcm. Dấu bằng xảy ra khi $a=b=c=1$


#658175 Marathon Phương trình và hệ phương trình VMF

Gửi bởi Ngockhanh99k48 trong 17-10-2016 - 10:38

Từ phương trình thứ nhất ta có $x^2+\frac{y^2}{4}+\frac{z^2}{9}+2.x.\frac{y}{2}.\frac{z}{3}=1$. Từ đây ta liên tưởng tới đẳng thức $X^2+Y^2+Z^2+2XYZ=1$. Như vậy tồn tại tam giác $ABC$ thỏa mãn: $x=\cos A, y=2\cos B, z=3\cos C$.
Như vậy, thay vào phương trình 2 ta có:
$3\cos A + 4\cos B + 5\cos C = 6 \sin \frac{A}{2} + 4\sin \frac{B}{2} + 2\sin \frac{C}{2}$ $\Leftrightarrow$ $2(\cos A+\cos C - 2\sin \frac{B}{2}) + (\cos A+\cos B - 2\sin\frac{C}{2})+ 3(\cos B+\cos C-2\sin\frac{A}{2})=0$. Sử dụng $\cos x +\cos y \leq 2cos\frac{x+y}{2}$ ta có dấu bằng phải xảy ra. Khi đó $\triangle ABC$ đều.


#658167 Tuần 3 tháng 10/2016 : Bài toán tiếp xúc với đường tròn cố định

Gửi bởi Ngockhanh99k48 trong 16-10-2016 - 23:48

Lời giải:
Vị tự tâm $A$ tỉ số 2 biến bài toán trên trở thành bài toán sau: Cho $\triangle ABC$ ngoại tiếp $(I)$, nội tiếp $(O)$ và đường tròn Mixtilinear- nội tiếp góc $\widehat{BAC}$ là $(X)$. $(X)$ tiếp xúc $AB, AC$ tại $F, E$. $BI, CI$ thứ tự cắt $(O)$ tại điểm thứ hai $P, Q$. $AP, AQ$ thứ tự cắt $EF$ tại $S, R$. Gọi $K, L$ thứ rự trên $XF, XE$ sao cho $SA \perp SK$ và $RA \perp RL$. $d$ là đường thẳng bất kì qua $A$. Gọi $U, V$ là hình chiếu của $K, L$ trên $d$. Chứng minh trục đẳng phương của $(UV)$ và $(O)$ tiếp xúc với $(X)$.
Nghịch đảo cực $A$, phương tích bất kì biến điểm $X$ thành điểm có kí hiệu $X'$.
Ta có $E', F'$ là tiếp điểm của đường tròn bàng tiếp góc $A$ là $(I')$ của $\triangle AB'C'$. Do $Q$ là tâm ngoại tiếp $\triangle AIB$ nên $R = EF \cap (AIB)$. Suy ra $R'=(AI') \cap I'B'$. Tương tự thì $S'= (AI') \cap I'C'$. Do $L= (AER) \cap XE$ nên $L' = E'R' \cap (AX'E')$. Để ý rằng $X'$ là tâm mixtilinear bàng tiếp góc $\widehat{BAC}$ của $\triangle AB'C'$ nên $\widehat{AX'E'} = 90^{\circ}$ hay $L'$ là hình chiếu của $A$ trên $E'R'$. Tương tự $K'$ là hình chiếu của $A$ trên $F'S'$. Từ đó $U', V'$ thứ tự là giao của $d$ với $E'R', F'S'$. Để ý rằng nếu tiếp điểm của $(I')$ với $B'C'$ là $D'$ thì $D'=E'R' \cap F'S'$. Giả sử $(UV)$ với $(ABC)$ cắt nhau tại hai điểm $Z, T$. Khi đó ảnh của trục đẳng phương của $(UV)$ và $(ABC)$ chính là đường tròn $(AZ'T')$. Do $(AZ'T')$ tiếp xúc với $(I')$ theo ELMO 2016 poblem 6  nên ta có đpcm.